Tuyển chọn các bài bất đẳng thức hay

94 65 0
Tuyển chọn các bài bất đẳng thức hay

Đang tải... (xem toàn văn)

Tài liệu hạn chế xem trước, để xem đầy đủ mời bạn chọn Tải xuống

Thông tin tài liệu

Tạ Minh Hoằng Nguyễn Huy Tùng Tuyển tập toán Tháng 11 năm 2010 Tài liệu thực mục đích học tập Bất thao tác, trao đổi tài liệu mục đích thương mại phải đồng ý tác giả ii Mục lục Mục lục iii Lời cảm ơn iv Một số kết ký hiệu 1.1 Một số kết 1.2 Các ký hiệu 1 Tuyển tập toán 2.1 Đề toán 2.2 Lời giải 5 14 Tài liệu tham khảo 90 iii Lời cảm ơn Chắc chắn tuyển tập hồn thành khơng có giúp đỡ từ người bạn Họ trực tiếp động viên chúng tơi thực hiện, góp ý để tuyển tập cách tốt toán bất đẳng thức Xin chân thành cảm ơn hai anh sau giúp nhiều việc thực tuyển tập Nguyễn Văn Dũng - Giảng viên Học Viện Kỹ thuật Quân Hà Nội; Võ Quốc Bá Cẩn - Sinh viên Đại học Y Dược Cần Thơ Chương Một số kết ký hiệu 1.1 Một số kết Trong phần liệt kê kết ký hiệu sử dụng Các chứng minh kết bạn tìm thấy tài liệu tham khảo mà ghi cuối tuyển tập (Bất đẳng thức AM – GM) Với số thực không âm a1 , a2 , , an , ta ln có a1 + a2 + + an √ ≥ n a1 a2 an n Đẳng thức xảy a1 = a2 = = an (Bất đẳng thức AM – GM suy rộng) Với số thực không âm x1 , x2 , , xn số thực dương α1 , α2 , , αn có tổng ta ln có α1 x1 + α2 x2 + + αn xn ≥ x1α1 x2α2 xnαn Đẳng thức xảy a1 = a2 = = an (Bất đẳng thức Cauchy – Schwarz) Cho hai số thực a1 , a2 , , an b1 , b2 , , bn Khi đó, ta ln có (a21 + a22 + + a2n )(b21 + b22 + + b2n ) ≥ (a1 b1 + a2 b2 + + an bn )2 Đẳng thức xảy tồn số thực k cho = bki với i = 1, 2, , n (Bt ng thc Hăolder) Vi m dãy số không âm xi j (i = 1, 2, , m, j = 1, 2, , n) p1 , p2 , , pn > thỏa mãn p1 + p2 + + pn = ta có m ∏ i=1 n pi n ≥ ∑ xi j m ∑ ∏ xipj i j=1 i=1 j=1 Đẳng thức xảy m dãy số tương ứng tỷ lệ (Bất đẳng thức Chebyshev) Giả sử a1 , a2 , , an b1 , b2 , , bn hai số thực Chương Một số kết ký hiệu 1.1 Một số kết (i) Nếu hai dãy đơn điệu chiều a1 b1 + a2 b2 + + an bn ≥ (a1 + a2 + + an )(b1 + b2 + + bn ) n (ii) Nếu hai dãy đơn điệu ngược chiều a1 b1 + a2 b2 + + an bn ≤ (a1 + a2 + + an )(b1 + b2 + + bn ) n (Bất đẳng thức trung bình lũy thừa) Cho a1 , a2 , , an số thực không âm Đặt  r r r   a1 + a2 + + an r r>0 Ar = n  √ r a1 a2 an r=0 Khi Ar gọi trung bình lũy thừa bậc r a1 , a2 , , an có tính chất Ar ≥ As với r ≥ s ≥ (Bất đẳng thức Schur) Với số thực không âm a, b, c, r, ta ln có ar (a − b)(a − c) + br (b − c)(b − a) + cr (c − a)(c − b) ≥ Đẳng thức xảy a = b = c a = b, c = hốn vị Có hai kết thường sử dụng r = r = Với r = 1, ta có bất đẳng thức Schur bậc ba a(a − b)(a − c) + b(b − c)(c − a) + c(c − a)(c − b) ≥ Các dạng tương đương bất đẳng thức a3 + b3 + c3 + 3abc ≥ ab(a + b) + bc(b + c) + ca(c + a) (a + b + c)3 + 9abc ≥ 4(a + b + c)(ab + bc + ca) abc ≥ (a + b − c)(b + c − a)(c + a − b) a2 + b2 + c2 + 9abc ≥ 2(ab + bc + ca) a+b+c Với r = 2, ta có bất đẳng thức Schur bậc bốn a2 (a − b)(a − c) + b2 (b − c)(b − a) + c2 (c − a)(c − b) ≥ Dạng tương đương bất đẳng thức a4 + b4 + c4 + abc(a + b + c) ≥ ab(a2 + b2 ) + bc(b2 + c2 ) + ca(c2 + a2 ) 1.1 Một số kết Chương Một số kết ký hiệu (Bất đẳng thức Vornicu – Schur) Cho a ≥ b ≥ c số thực x, y, z hàm số không âm Xét bất đẳng thức sau x(a − b)(a − c) + y(b − c)(b − a) + z(c − a)(c − b) ≥ Bất đẳng thức tiêu chuẩn sau thỏa mãn x ≥ y (hoặc z ≥ y); x + z ≥ y; √ √ √ x + z ≥ y; ax ≥ by (hoặc cz ≥ by) với a ≥ b ≥ c ≥ 0; ax + cz ≥ by với a ≥ b ≥ c ≥ 0; √ √ √ ax + cz ≥ by với a ≥ b ≥ c ≥ 0; bz ≥ cy với a, b, c độ dài ba cạnh tam giác; x2 + y2 + z2 ≤ 2(xy + yz + zx) (Hàm lồi) Cho I khoảng R Một hàm f xác định I gọi lồi với a, b ∈ I α, β ≥ thỏa mãn α + β = 1, ta có α f (a) + β f (b) ≥ f (αa + β b) Nếu bất đẳng thức ngược chiều f gọi hàm lõm Nếu f khả vi I f lồi đạo hàm f hàm tăng Nếu f liên tục [a; b] có đạo hàm f (a, b), f lồi f ≥ 10 (Bất đẳng thức Jensen) Nếu a1 , a2 , , an số thực không âm cho a1 + a2 + + an = x1 , x2 , , xn số thực với hàm f lồi R ta ln có a1 f (x1 ) + a2 f (x2 ) + + an f (xn ) ≥ f (a1 x1 + a2 x2 + + an xn ) 11 (Bất đẳng thức Newton) Với số thực không âm a1 , a2 , , an , đặt n ∑ Sk = i1 0, ta ln có a2 + bc b2 + ca c2 + ab a2 + b2 + c2 + + ≥ + b2 + bc + c2 c2 + ca + a2 a2 + ab + b2 3(ab + bc + ca) Với a, b, c số thực không âm thỏa mãn ab + bc + ca > Hãy chứng minh 1 1 + + ≥ + b2 + bc + c2 c2 + ca + a2 a2 + ab + b2 3(ab + bc + ca) (a + b + c)2 Chương Tuyển tập toán 2.1 Đề toán Nếu a, b, c ≥ thỏa mãn (a + b)(b + c)(c + a) > a3 b3 c3 (a2 + b2 + c2 )3 + + ≥ b2 + 4bc + c2 c2 + 4ca + a2 a2 + 4ab + b2 2(a + b + c)(ab + bc + ca)2 Giả sử a, b, c số thực dương Hãy chứng minh a+b a2 + bc + c2 + b+c b2 + ca + a2 + c+a c2 + ab + b2 ≥ 27(ab2 + bc2 + ca2 + 3abc) (a + b + c)4 10 Với a, b, c ≥ thỏa mãn ab + bc + ca > ta ln có a3 b3 c3 a4 + b4 + c4 + + ≥ 2b2 − bc + 2c2 2c2 − ca + 2a2 2a2 − ab + 2b2 a3 + b3 + c3 11 Cho a, b, c ≥ thỏa mãn (a + b)(b + c)(c + a) > Chứng minh 4(a + b + c) 1 ∑ 4a + b + c + a2 + b2 + c2 + 3(ab + bc + ca) ≤ ∑ b + c 12 Nếu a, b, c số thực dương bc ca ab 2(a2 + b2 + c2 ) + ab + bc + ca + + ≥ (c + a)(a + b) (a + b)(b + c) (b + c)(c + a) 2(a2 + b2 + c2 + ab + bc + ca) 13 Cho a, b, c số thực không âm thỏa mãn khơng có hai số đồng thời Chứng minh a b c 3(a2 + b2 + c2 ) + + + ≥ b+c c+a a+b (a + b + c)2 14 Với a, b, c > thỏa mãn 3(a2 + b2 + c2 ) + ab + bc + ca = 12 ta ln có b c a √ +√ +√ ≤√ c+a a+b b+c 15 Cho số thực không âm a, b, c thỏa mãn ab + bc + ca > Chứng minh a b+c + b c+a + c a+b + a2 + b2 + c2 ≥ · ab + bc + ca 16 Cho số thực không âm a, b, c thỏa mãn (a + b)(b + c)(c + a) > Hãy chứng minh 1 + + ≥ 2 (b + c) (c + a) (a + b) 4(ab + bc + ca) (Iranian Mathematical Olympiad 1996) 17 Giả sử a, b, c số thực khơng âm, chúng khơng có hai số đồng thời Hãy chứng minh bất đẳng thức sau a2 + 2bc b2 + 2ca c2 + 2ab 2abc + + + ≥ (b + c)2 (c + a)2 (a + b)2 (a + b)(b + c)(c + a) Chương Tuyển tập toán 2.2 Lời giải Điều chứng tỏ P(a, b, c, d) ≥ P(t,t,t, d) Mặt khác, t ≤ P(t,t,t, d) − nên 148t d 148t (1 − 3t) = 3t + d + − = 3t + (1 − 3t)4 + − 27 27 27 27 27 2(1 − 3t)(13 − 19t)(1 − 4t)2 = ≥ 27 Từ ta có điều phải chứng minh Đẳng thức xảy a = b = c = d = a = b = c = 13 , d = hoán vị Nhận xét Ngồi ta, có số kết dạng Bài toán Cho a, b, c, d số thực không âm thỏa mãn a + b + c + d = Khi (1 + 3a)(1 + 3b)(1 + 3c)(1 + 3d) ≤ + 353abcd; (i) abc + bcd + cda + dab ≤ (ii) 176 + abcd; 27 27 ab + ac + ad + bc + bd + cd ≤ (iii) 32 + abcd; 3 9(abc + bcd + cda + dab) ≤ ab + ac + ad + bc + bd + cd + 48abcd (iiii) Bạn đọc thử sức với chúng nhé! 70 Giả sử a, b, c, d số thực dương có tổng Hãy chứng minh abc + bcd + cda + dab + (abc)2 + (bcd)2 + (cda)2 + (dab)2 ≤ Lời giải Giả sử a ≥ b ≥ c ≥ d đặt t = a+b+c Ta chứng minh hai bất đẳng thức sau abc + bcd + cda + dab ≤ t + 3dt , (2.9) (abc)2 + (bcd)2 + (cda)2 + (dab)2 ≤ t + 3d 2t (2.10) Bất đẳng thức (2.9) hiển nhiên theo AM – GM Vậy ta cần xét bất đẳng thức (2.10) đủ Bất đẳng thức tương đương với (t − abc)(t + abc) + 3d 2t(t − abc) ≥ d (a2 b2 + b2 c2 + c2 a2 − 3tabc) Ta có đẳng thức sau t − abc = a+b+c ∑ (a + 4b + 4c)(a − b)(a − c) − abc = a,b,c 27 , a2 b2 + b2 c2 + c2 a2 − 3tabc = a2 b2 + b2 c2 + c2 a2 − abc(a + b + c) = bc(a − b)(a − c) + ca(b − c)(b − a) + ab(c − a)(c − b), nên bất đẳng thức viết lại thành x(a − b)(a − c) + y(b − c)(b − a) + z(c − a)(c − b) ≥ 0, 76 2.2 Lời giải Chương Tuyển tập toán với x = (t + abc + 3d 2t)(a + 4b + 4c) − 27bcd = (t − abc)(a + 4b + 4c) + d (a2 + 7bc) + 4d (b − c)2 + 2bc(a2 + 4ab + 4ac − 9d ) ≥ 0, biểu thức y, z tương tự Ta có ax − by = (t + abc + 3d 2t)(a2 − b2 + 4ac − 4bc) ≥ 0, nên kết hợp với a − c ≥ a(b−c) b , ta x(a − b)(a − c) + y(b − c)(b − a) + z(c − a)(c − b) = (a − b)[x(a − c) − y(b − c)] + z(c − a)(c − b) (a − b)(b − c)(ax − by) + z(c − a)(c − b) ≥ ≥ b Vậy bất đẳng thức (2.10) chứng minh Sử dụng (2.9) (2.10), ta đưa điều phải chứng minh trở thành t + 3dt + t + 3d 2t ≤ Thay d = − 3t thực biến biến đổi, ta có bất đẳng thức tương đương với (4t + 3t + 4t + − 7t )(t − 1)2 ≥ 0, bất đẳng thức hiển nhiên t < 43 Đẳng thức xảy a = b = c = d = 71 Cho số dương a, b, c, d có tích Khi đó, ta có (a − 1)(a − 2) + (b − 1)(b − 2) + (c − 1)(c − 2) + (d − 1)(d − 2) ≥ Lời giải Không tính tổng quát giả sử a ≥ b ≥ c ≥ d Lúc này, ta có abc ≥ ≥ d Để chứng minh bất đẳng thức cho, ta cần có bất đẳng thức sau (a − 1)(a − 2) + (b − 1)(b − 2) + (c − 1)(c − 2) ≥ √ abc − √ abc − Thay a, b, c a3 , b3 , c3 , ta có bất đẳng thức tương đương với a6 + b6 + c6 − 3a2 b2 c2 ≥ 3(a3 + b3 + c3 − 3abc) Sử dụng đẳng thức quen thuộc x3 + y3 + z3 − 3xyz = (x + y + z) ∑(x − y)(x − z), ta viết lại bất đẳng thức thành S = x(a − b)(a − c) + y(b − c)(b − a) + z(c − a)(c − b) ≥ 0, 77 (2.11) Chương Tuyển tập toán 2.2 Lời giải x = (a2 + b2 + c2 )(a + b)(a + c) − 3(a + b + c) ≥ a2 (ab + bc + ca) + (ab + bc + ca)2 − 3abc(a + b + c) ≥ 0, biểu thức y, z tương tự Dễ thấy y ≥ z, S ≥ y(a − b)(a − c) + y(b − c)(b − a) = y(a − b)2 ≥ Vậy (2.11) chứng minh Sử dụng nó, ta đưa toán Đặt √ abc − + (d − 1)(d − 2) ≥ √ abc − √ abc = t thay d = t3 vào, ta cần chứng minh −1 t3 3(t − 1)(t − 2) + − ≥ t3 Khơng khó khăn, ta viết lại thành (t − 1)2 (3t − 3t − t + t + 3t + 2t + 1) ≥ 0, 3t − 3t − t + t + 3t + 2t + = t (3t − 1)(t − 1) + 3t + 2t + > (vì t ≥ 1) Đẳng thức xảy a = b = c = d = 72 Cho a, b, c, d số thực không âm cho chúng khơng có hai số đồng thời Chứng minh 1 1 1 243 + + + + + ≥ a3 + b3 a3 + c3 a3 + d b3 + c3 b3 + d c3 + d 2(a + b + c + d)3 Lời giải Đặt P(a, b, c, d) = V T −V P Giả sử d = min{a, b, c, d}, ta chứng minh P(a, b, c, d) đạt giá trị nhỏ d = Để chứng minh điều này, ta cần có bất đẳng thức sau d d d P(a, b, c, d) ≥ P a + , b + , c + , 3 Ta có a3 + d ≥ a3 + b3 b3 + c3 a+ d 3 , b3 + d ≥ a+ d + ≥ b+ d + c+ d 3 b+ , d 3 ≥ b+ d 3 , , a3 + c3 (2.12) c3 + d ≥ c + d3 ≥ a+ d + c + d3 3 , , 243 243 = 2(a + b + c + d) a + d3 + b + d3 + c + d3 Cộng tất đánh giá lại ta thu (2.12) Vậy ta cần chứng minh 1 1 1 243 + 3+ 3+ + + ≥ 3 3 a b c b +c c +a a +b 2(a + b + c)3 78 2.2 Lời giải Chương Tuyển tập toán Sử dụng bất đẳng thức AM – GM, ta có 1 + + b3 c3 b3 + c3 2V T = ∑ ≥ 3∑ ≥ 3∑ 24 ∑ b3 c3 (b3 + c3 ) = 3∑ 3bc+b2 −bc+c2 b3 c3 (b2 − bc + c2 )(b + c) = 24 ∑ (b + c) , (b + c)3 ≥ 24 · (b + c) (a + b)(b + c)(c + a) 243 ≥ 24 · = a+b+b+c+c+a (a + b + c)3 Suy 2 2 2 243 + + + + + ≥ a3 b3 c3 b3 + c3 c3 + a3 a3 + b3 (a + b + c)3 Từ đây, chia hai vế cho 2, ta thu kết cần chứng minh Với giả thiết trên, đẳng thức có a = b = c d = Nhận xét Chú ý số k tốt cho bất đẳng thức 1 1 1 k + + + + + ≥ a3 + b3 a3 + c3 a3 + d b3 + c3 b3 + d c3 + d (a + b + c + d)3 k = 243 Đây bất đẳng thức chặt khó 73 Với số thực không âm a, b, c, d thỏa mãn abc + bcd + cda + dab > ta ln có 1 1 1 81 + + + + + ≥ a2 + b2 a2 + c2 a2 + d b2 + c2 b2 + d c2 + d 2(a + b + c + d)2 Lời giải Tương tự toán trước, đặt P(a, b, c, d) = V T − V P, ta chứng minh P(a, b, c, d) đạt giá trị nhỏ d = với a ≥ b ≥ c ≥ d Khi đó, đánh sau d d P(a, b, c, d) ≥ P a, b + , c + , 2 Thật vậy, sau vài biến đổi, ta viết lại sau d(4b + d) d(4c + d) + (a2 + b2 )(4a2 + 4b2 + 4bd + d ) (a2 + c2 )(4a2 + 4c2 + 4cd + d ) d(2b + 2c + d) d2 + − (b + c2 )(2b2 + 2c2 + 2bd + 2cd + d ) a2 (a2 + d ) 2d 2d + + ≥ (b + d )(2b + d)2 (c2 + d )(2c + d)2 79 Chương Tuyển tập toán 2.2 Lời giải Bất đẳng thức thực chất hệ bất đẳng thức sau 4b + d d ≥ , (a2 + b2 )(4a2 + 4b2 + 4bd + d ) 4a2 (a2 + d ) 4c + d d ≥ 2 , 2 2 (a + c )(4a + 4c + 4cd + d ) 4a (a + d ) d 2b + 2c + d ≥ 2 2 2 (b + c )(2b + 2c + 2bd + 2cd + d ) 2a (a + d ) Bất đẳng thức thứ tương đương với 8a2 bd(b − d) + 4bd (a2 − bd) + d (a2 − b2 ) + 4b(a4 − b3 d) + 2a2 (d + 6a2 b) ≥ Hoàn toàn tương tự với bất đẳng thức thứ hai 8a2 cd(c − d) + 4cd (a2 − cd) + d (a2 − c2 ) + 4c(a4 − c3 d) + 2a2 (d + 6a2 c) ≥ Bất đẳng thức thứ ba dạng tương đương 2b(a4 − b3 d) + 2c(a4 − c3 d) + 2b(a3 − bc2 d) + 2c(a3 − b2 cd) + 2d (a2 − bc)(b + c) + d (2b + d)(a2 − b2 ) + d (2c + d)(a2 − c2 ) + 2a4 d ≥ Các đánh giá chứng tỏ ta cần chúng minh P(a, b, c, d) ≥ với d = Khi đó, bất đẳng thức trở thành 1 1 1 81 + + + + + ≥ a2 b2 c2 b2 + c2 c2 + a2 a2 + b2 2(a + b + c)2 Sử dụng bất đẳng thức AM – GM Cauchy – Schwarz, ta có 1 1 + ≥ + ∑ a2 ∑ b2 + c2 ∑ 2bc b2 + c2 = 4∑ ≥∑ 2 2bc + b + c (b + c)2 ≥ ≥ 1 + + b+c c+a a+b (2.13) (b + c) + (c + a) + (a + b) = 27 , (a + b + c)2 = 27 2(a + b + c)2 1 + 2+ 2 a b c ≥ 1 + + a b c ≥ a+b+c (2.14) Cộng (2.13) (2.14) lại, ta có điều phải chứng minh.Với a ≥ b ≥ c ≥ d, đẳng thức có a = b = c d = 80 2.2 Lời giải Chương Tuyển tập toán 74 Cho a, b, c, d, e số thực dương Chứng minh c 1+ a+b d 1+ b+c e 1+ c+d a 1+ d +e b 1+ e+a ≥ Lời giải Khơng tính tổng quát giả sử a + b + c + d + e = Khi đó, ta thực biến đổi bất đẳng thức cho sau (a + b + c)(b + c + d)(c + d + e)(d + e + a)(e + a + b) ≥ (a + b)(b + c)(c + d)(d + e)(e + a) a+b+c b+c+d c+d +e d +e+a e+a+b · · · · ≥ d +e e+a a+b b+c c+d −1 d +e −1 e+a −1 a+b −1 b+c 3 , , −1 ≥ c+d Để ý với x, y > thỏa mãn x + y ≤ ta có −1 x −1 − −1 y x+y = (x − y)2 (1 − x − y) ≥ xy(x + y)2 Từ đánh giá ta suy −1 d +e −1 ≥ a+b −1 d +e+a+b 2 −1 1−c = Một cách tương tự −1 e+a −1 ≥ b+c −1 1−d −1 a+b −1 ≥ c+d −1 1−e −1 b+c −1 ≥ d +e −1 1−a −1 c+d −1 ≥ e+a −1 1−b , , , Nhân năm bất đẳng thức chiều cho ta 1 1 −1 −1 −1 −1 −1 d +e e+a a+b b+c c+d 2 2 ≥ −1 −1 −1 −1 −1 1−a 1−b 1−c 1−d 1−e Vậy để chứng minh bất đẳng thức cho, ta cần chứng minh −1 1−a −1 1−b −1 1−c 81 −1 1−d −1 ≥ 1−e Chương Tuyển tập toán 2.2 Lời giải Lấy logarith nêpe hai vế xét hàm số f (x) = ln x ∈ [0, 1], ta có 1−x − = ln(1 + x) − ln(1 − x) với 1 + 1+x 1−x −1 −1 f (x) = + ≥ + = (1 + x)2 (1 − x)2 (1 + x)2 (1 + x)2 f (x) = Suy f (x) hàm lồi, bất đẳng thức Jensen cho ta a+b+c+d +e f (a) + f (b) + f (c) + f (d) + f (e) ≥ f = 5f = ln Bài toán giải Đẳng thức xảy a = b = c = d = e Nhận xét Chúng ta có kết tổng quát Bài toán Nếu a1 ≥ a2 ≥ ≥ an > 1+ a3 a1 + a2 1+ a4 a2 + a3 ··· 1+ a2 an + a1 n ≥ 75 Giả sử a, b, c, d, e số thực khơng âm có tổng Chứng minh 729 (a2 + b2 )(b2 + c2 )(c2 + d )(d + e2 )(e2 + a2 ) ≤ Lời giải Do vai trò hốn vị vòng quanh biến a, b, c, d, e nên ta giả sử e = min{a, b, c, d, e} Đặt P(a, b, c, d, e) vế trái bất đẳng thức cho Ta có a2 + b2 ≤ a + nên e 2 e d + e2 ≤ d + e2 + a2 ≤ a + , + b2 , e 2 , c2 + d ≤ c2 + d + e 2 , e e P(a, b, c, d, e) ≤ P a + , b, c, d + , 2 Từ suy ta cần chứng minh bất đẳng thức ban đầu với e = Khi đó, ta có a+b+c+d = bất đẳng thức trở thành Q(a, b, c, d) = a2 d (a2 + b2 )(b2 + c2 )(c2 + d ) ≤ 729 (2.15) Do vai trò b, c nhau, giả sử b ≤ c Bây giờ, ta lại có a2 (a2 + b2 ) ≤ a2 + ab + nên b2 = a+ b 2 , b2 + c2 ≤ c + b b Q(a, b, c, d) ≤ Q a + , 0, c + , d 2 82 b 2 , 2.2 Lời giải Chương Tuyển tập toán Từ suy (2.15) chứng minh b = Lúc a + c + d = ta cần có a4 c2 d (c2 + d ) ≤ 729 Sử dụng bất đẳng thức AM – GM, ta có 2a4 c2 d (c2 + d ) = a4 · cd · 2cd(c2 + d ) c+d ≤ a4 = 729 ≤ 729 = 729 a a 2 2cd + c2 + d 2 = 4a4 c+d c+d c+d c+d + a2 + c+d + + 10 10 a+c+d = 729 Chứng minh hoàn tất Đẳng thức xảy (a, b, c, d, e) = 2, 0, 23 , 32 , hoán vị tương ứng 76 Với a, b, c, d, e số thực dương có tổng 5, chứng minh abc + bcd + cde + dea + eab ≤ Lời giải Khơng tính tổng qt, giả sử a = min{a, b, c, d, e} Khi đó, sử dụng bất đẳng thức AM – GM, ta có abc + bcd + cde + dea + eab = cd(b + e − a) + a(b + d)(c + e) ≤ c+d +b+e−a = − 2a 3 +a b+d +c+e +a 5−a Mặt khác, ta lại có − 2a 5− 3 5−a −a 2 = 5(1 − a)2 (2 + a) ≥ 0, 108 nên kết hợp với trên, ta suy kết cần chứng minh Đẳng thức xảy tất biến a, b, c, d, e 77 Cho sáu số thực dương a, b, c, x, y, z Chứng minh (a + b + c)(x + y + z) ax by cz ≥ + + a+b+c+x+y+z a+x b+y c+z 83 Chương Tuyển tập toán 2.2 Lời giải Lời giải Ta thực biến đổi bất đẳng thức cho sau a + b + c + x + y + z (a + b + c)(x + y + z) − a+b+c+x+y+z ax by a+x b+y c+z c+z − − − ≤ + + , a+x b+y cz (a + b + c − x − y − z)2 (a − x)2 (b − y)2 (c − z)2 ≤ + + , a+b+c+x+y+z a+x b+y c+z hiển nhiên theo bất đẳng thức Cauchy – Schwarz VP = [(a − x) + (b − y) + (c − z)]2 (a − x)2 (b − y)2 (c − z)2 ≤ + + = V T (a + x) + (b + y) + (c + z) a+x b+y c+z Bài toán giải 78 Với hai tam giác ABC A B C ta có √ a (b + c − a) + b (c + a − b) + c (a + b − c) ≥ 3SS Lời giải Đặt x = p − a, y = p − b, z = p − c, x = p − a , y = p − b , z = p − c ta có x, y, z, x , y , z số dương bát đẳng thức cần chứng minh trở thành 2x(y + z ) + 2y(z + x ) + 2z(x + y ) ≥ 4 9xyz(x + y + z)x y z (x + y + z ), x(y + z ) + y(z + x ) + z(x + y ) + x (y + z) + y (z + x) + z (x + y) ≥ 4 9xyz(x + y + z)x y z (x + y + z ) Áp dụng bất đẳng thức AM – GM, ta có x(y + z ) + y(z + x ) + z(x + y ) ≥ 3 xyz(x + y )(y + z )(z + x ), x (y + z) + y (z + x) + z (x + y) ≥ 3 x y z (x + y)(y + z)(z + x) Sử dụng hai kết này, ta x(y + z ) + y(z + x ) + z(x + y ) + x (y + z) + y (z + x) + z (x + y) ≥ 3 xyz(x + y )(y + z )(z + x ) + 3 x y z (x + y)(y + z)(z + x) ≥ 6 xyz(x + y)(y + z)(z + x)x y z (x + y )(y + z )(z + x ) Mặt khác, từ ab+bc+ca ≥ √ √ 9(a+b)(b+c)(c+a) ≥ 8(a+b+c)(ab+bc+ca) ta suy 3(a + b)(b + c)(c + a) ≥ abc(a + b + c)3/2 , xyz(x + y)(y + z)(z + x)x y z (x + y )(y + z )(z + x ) ≥ 64(xyz)3/2 (x + y + z)3/2 (x y z )3/2 (x + y + z )3/2 27 84 3abc(a + b + c) 2.2 Lời giải Chương Tuyển tập toán Kết hợp với trên, ta dễ dàng suy x(y + z ) + y(z + x ) + z(x + y ) + x (y + z) + y (z + x) + z (x + y) ≥ 6 xyz(x + y)(y + z)(z + x)x y z (x + y )(y + z )(z + x ) 64(xyz)3/2 (x + y + z)3/2 (x y z )3/2 (x + y + z )3/2 27 = 9xyz(x + y + z)x y z (x + y + z ) ≥6 Đây điều phải chứng minh Đẳng thức xảy tam giác ABC A B C 79 Cho a1 , a2 , , an (n ≥ 3) số thực khơng âm có tổng Hãy chứng minh n √ n n2 √ √ a ≤ i ∑ ∑ + n+1 i=1 i=1 Lời giải Theo bất đẳng thức Cauchy – Schwarz √ √ √ a1 + a2 + + an ≤ 1 n a1 + + + n n n−1 n + a2 + + an n + − a1 n = n a1 + = (na1 + n − 1)(n + − na1 ) n Từ suy n √ ∑ i=1 √ ≤ + a1 = (na1 + n − 1)(n + − na1 ) n(1 + a1 ) n + − na1 − 2n + n(1 + a1 ) Thiết lập đánh giá tương tự, ta chứng minh bất đẳng thức mạnh n ∑ i=1 n + − nai − 2n + n2 ≤√ n(1 + ) n+1 85 Chương Tuyển tập toán 2.2 Lời giải Lại sử dụng bất đẳng thức Cauchy – Schwarz ta có n ∑ n + − nai − i=1 2n + ≤ n(1 + ) n n ∑ n + − nai − i=1 2n + n(1 + ) n i=1 + = n2 (n + 1) − (2n + 1) ∑ ≤ n2 (n + 1) − n2 (2n + 1) n n + ∑ i=1 = n2 (2n + 1) n2 =√ n+1 n+1 n2 (n + 1) − Bài toán giải Đẳng thức xảy = 1n , ∀i = 1, 2, , n 80 Chứng minh với k = n − n  ∑  (n − 1) i=1 ≥ n2 + k  n  i=1 ∑ a j n n ∑ ∑ i=1  a2i  − 2 , i< j a1 , a2 , , an (n ≥ 3) số thực dương Lời giải Bất đẳng thức cho tương đương với bất đẳng thức dãy sau n n ∑ i< j a j + a j (n − 1)2 ∑ a2i i=1 + n ≥ n2 − 2n + + n , ∑ a j i< j n n ∑ i< j a j + + + 2n − n2 ≥ n2 − 2n + + a j (n − 1)2 ∑ a2i i=1 n ∑ a j i< j n n (ai + a j )2 ≥ 2(n − 1)2 + a a i j i< j (n − 1)2 ∑ a2i ∑ i=1 n ∑ a j i< j n (ai + a j )2 ∑ a j ≥ i< j (n − 1)2 n ∑ i=1 n ∑ a j i< j 86 , , 2.2 Lời giải Chương Tuyển tập tốn Ta thấy bất đẳng thức hiển nhiên theo Cauchy – Schwarz n i< j VT ≥ ∑ i=1 = n n (n − 1)2 ∑ (ai + a j ) = V P n ∑ a j ∑ a j i< j i

Ngày đăng: 14/06/2020, 16:56

Từ khóa liên quan

Mục lục

  • Mục lục

  • Lời cảm ơn

  • Một số kết quả và các ký hiệu

    • Một số kết quả

    • Các ký hiệu

    • Tuyển tập các bài toán

      • Đề toán

      • Lời giải

      • Tài liệu tham khảo

Tài liệu cùng người dùng

Tài liệu liên quan